Page LayoutMy page breaks are being ignored!

Information and discussion about page layout specific issues (e.g. header and footer lines, page formats, page numbers).
yaelcita
Posts: 6
Joined: Sat May 29, 2010 11:46 pm

My page breaks are being ignored!

Post by yaelcita »

I've had this happen a couple of times now. I have a document that has no issues, and for the first 3 or 4 pages everything looks perfectly fine, but then on the next page the bottom margin is completely ignored, and whatever I'm writing continues there and even out of the page, for about 6cm (see below), where I can't see it.

So far, I've just added vertical space before the part that was being written on the margin, and that has solved the issue. (That's how I know it continues writing for about 6cm). Of course, it's not a good solution, because I have to change that every time I change something before the part where the problem is occurring, but I needed something quick. Adding a manual pagebreak also works, but then a chunk at the bottom of my page is blank.

This has happened both with text and equations, and I think once with a figure. Also, once it happens in one page, it seems to happen in all the ones after it, although my documents are short enough that I don't know for sure.

Oh, and I have used the same preamble (copied and pasted) in all of my documents where this has happened, so there might be something there, but I don't know what. This is what the preamble looks like:

Code: Select all

\documentclass[a4paper,11pt,answers]{exam}
\printanswers
\qformat{\textbf{Problem \thequestion}\hfill}
\noaddpoints

\usepackage[pdftex]{graphicx}

\renewcommand{\solutiontitle}{\textbf{Solution:}\par\noindent}
\renewenvironment{TheSolution}
  {
    \vspace{\parskip}
      \leftskip=0pt
    \rightskip=0pt
    \solutiontitle
    \ignorespaces
  }

\usepackage{amsmath}
\usepackage{amssymb}

\usepackage{enumerate}
\usepackage{paralist}
\usepackage{verbatim}

\usepackage[left=2cm,top=2cm,right=3cm,nohead,nofoot]{geometry}
\footskip=1cm

\author{....}
\title{....}
\date{}

\begin{document}
Any help would be greatly appreciated!!!!
Last edited by yaelcita on Fri Jun 04, 2010 9:07 am, edited 3 times in total.

Recommended reading 2024:

LaTeXguide.org • LaTeX-Cookbook.net • TikZ.org

NEW: TikZ book now 40% off at Amazon.com for a short time.

And: Currently, Packt sells ebooks for $4.99 each if you buy 5 of their over 1000 ebooks. If you choose only a single one, $9.99. How about combining 3 LaTeX books with Python, gnuplot, mathplotlib, Matlab, ChatGPT or other AI books? Epub and PDF. Bundle (3 books, add more for higher discount): https://packt.link/MDH5p

meho_r
Posts: 823
Joined: Tue Aug 07, 2007 5:28 pm

My page breaks are being ignored!

Post by meho_r »

Hi. Please use:

Code: Select all

[code]your_code_here
[/code]
when writing a code. The code/preamble you provided works fine on my machine, no matter what is the number of pages (providing that exam class is the one that comes with TeXLive2009). However, you haven't provided enough infos to thoroughly test the issue. Please, try to create a complete MWE (Minimal Working Example) which shows the issue, so people here can test it. Also, information about TeX installation would be useful (put the command \listfiles at the very beginning of your document, compile it and post here file list which will be displayed at the end of your .log file).
yaelcita
Posts: 6
Joined: Sat May 29, 2010 11:46 pm

My page breaks are being ignored!

Post by yaelcita »

That's the issue, though... I don't know when this happens. It seems to be random. Sometimes I write an entire document without having this happen, sometimes it happens on half of my pages, sometimes on one only.... I see no consistency between the instances when it happens. I'm completely confused! In fact, I just tried to see if I could get rid of all but the part where one of my documents does that, but I can't. If I remove anything before the problem part, it works perfectly... So, here is all of it! The problem shows up in the last page. Note that when I remove the figure it works, so you will need to include some figure in there. I don't think it matters what, because it does this with many different figures.

Also, here's the output of listfiles:

Code: Select all

*File List*
    exam.cls    2008/07/19 Version 2.3 by Philip Hirschhorn
  ifthen.sty    2001/05/26 v1.1c Standard LaTeX ifthen package (DPC)
 article.cls    2005/09/16 v1.4f Standard LaTeX document class
  size11.clo    2005/09/16 v1.4f Standard LaTeX file (size option)
graphicx.sty    1999/02/16 v1.0f Enhanced LaTeX Graphics (DPC,SPQR)
  keyval.sty    1999/03/16 v1.13 key=value parser (DPC)
graphics.sty    2006/02/20 v1.0o Standard LaTeX Graphics (DPC,SPQR)
    trig.sty    1999/03/16 v1.09 sin cos tan (DPC)
graphics.cfg    2007/01/18 v1.5 graphics configuration of teTeX/TeXLive
  pdftex.def    2008/09/08 v0.04l Graphics/color for pdfTeX
 amsmath.sty    2000/07/18 v2.13 AMS math features
 amstext.sty    2000/06/29 v2.01
  amsgen.sty    1999/11/30 v2.0
  amsbsy.sty    1999/11/29 v1.2d
  amsopn.sty    1999/12/14 v2.01 operator names
 amssymb.sty    2002/01/22 v2.2d
amsfonts.sty    2001/10/25 v2.2f
enumerate.sty    1999/03/05 v3.00 enumerate extensions (DPC)
paralist.sty    2002/03/18 v2.3b Extended list environments (BS)
verbatim.sty    2003/08/22 v1.5q LaTeX2e package for verbatim enhancements
geometry.sty    2008/12/21 v4.2 Page Geometry
   ifpdf.sty    2007/12/12 v1.6 Provides the ifpdf switch (HO)
  ifvtex.sty    2008/11/04 v1.4 Switches for detecting VTeX and its modes (HO)
geometry.cfg
supp-pdf.tex
    umsa.fd    2002/01/19 v2.2g AMS font definitions
    umsb.fd    2002/01/19 v2.2g AMS font definitions
problem3.png    Graphic file (type png)
***********
Thanks for any and all help!!!

Code: Select all

\documentclass[a4paper,11pt,answers]{exam}
\usepackage[pdftex]{graphicx}
\printanswers
\qformat{\textbf{Problem \thequestion}\hfill}
\noaddpoints

\renewcommand{\solutiontitle}{\textbf{Solution:}\par\noindent}
\renewenvironment{TheSolution}
  {
    \vspace{\parskip}
      \leftskip=0pt
    \rightskip=0pt
    \solutiontitle
    \ignorespaces
  }

\usepackage{amsmath}
\usepackage{amssymb}

\usepackage{enumerate}
\usepackage{paralist}
\usepackage{verbatim}

\newcommand{\dis}{\displaystyle}
\usepackage[left=2cm,top=2cm,right=3cm,nohead,nofoot]{geometry}
\footskip=1cm

\author{kjhkjh}

\title{PHYSICS}

\date{}

\begin{document}
\maketitle

\centering{\LARGE{Homework 6 Solutions}}

\begin{questions}

\question \begin{inparaenum} [\bfseries(a)] \item Find the normal frequencies for small oscillations of the double pendulum for arbitrary values of the masses and lengths.

\item Check that your answers are correct for the special case that $m_1=m_2$ and $L_1=L_2$.

\item Discuss the limit that $m_2\rightarrow 0$
\end{inparaenum}

\begin{solution}
	\begin{inparaenum}[\bfseries(a)]

	\item The entire first part of this problem is actually worked out in Taylor as an example (pages 431-434), so I won't re-derive it here. Instead, I'll take over from where Taylor left off.
	
	The equation of motion is $\mathbf{M}\boldsymbol{\ddot{\phi}}=-\mathbf{K}\boldsymbol{\phi}$, where $\mathbf{M}$ and $\mathbf{K}$ are given by equation 11.44 in Taylor:
	\[ \mathbf{M}= 
	\begin{pmatrix}
		(m_1+m_2)L_1^2&m_2L_1L_2\\
		m_2L_1L_2&m_2L_2^2
	\end{pmatrix}, \hspace{1cm}
	\mathbf{K}=
	\begin{pmatrix}
		(m_1+m_2)gL_1&0\\
		0& m_2gL_2
	\end{pmatrix}
	\]
	
	The normal frequencies are found by proposing oscillatory solutions, writing them as the real part of an exponential, plugging them back into the equation of motion, and then solving $(\mathbf{K}-\omega^2\mathbf{M})\mathbf{a}=0$, which only has solutions when det$(\mathbf{K}-\omega^2\mathbf{M})=0$. So we just have to find the values of $\omega$ that satisfy that determinant equation. I did this entirely in Mathematica, because it's ugly and messy. I invite you to do the same. The answer is:
	$$ \omega^2=\frac{g(L_1+L_2)(m_1+m_2)\pm g\sqrt{(m_1+m_2)\left[(L_1-L_2)^2m_1+(L_1+L_2)^2m_2\right]}}{2L_1L_2m_1}$$
	which is just scary.
	
	Notice we can re-write the equation for $\mathbf{a}$ to make it more clear that it's just an eigenvalue equation: 
	\begin{eqnarray*}
		(\mathbf{K}-\omega^2\mathbf{M})\mathbf{a}&=&0\\
		\mathbf{K\, a}&=& \omega^2\mathbf{M \,a}\\
		\mathbf{M}^{-1}\mathbf{K \,a}&=&\omega^2\mathbf{a}
	\end{eqnarray*}
	so another way to find our values of $\omega$ is to see that they are just the eigenvalues of that matrix.
   
   	\item When $m_1=m_2$ and $L_1=L_2$, the first term in the root cancels, and the rest is:
   	$$ \omega^2=\frac{g2L\cdot2m\pm g\sqrt{2m^2\,4L^2}}{2L^2m}=\frac{g2L\cdot2m\pm g\,2m\cdot L\sqrt{2}}{2L^2m}=\frac{g}{L}\left(2\pm\sqrt{2}\right)$$
	which is exactly equation 11.47 where Taylor describes that example.
	
	\item In the case when $m_2=0$ we get:
	$$\omega^2=\frac{g(L_1+L_2)m_1\pm g\sqrt{m_1^2(L_1-L_2)^2}}{2L_1L_2m_1}=\frac{g\left(L_1+L_2\pm(L_1-L_2)\right)}{2L_1L_2}=\frac{g}{L_{1,2}}$$
   which is what we would expect: the pendulum has been reduced to a simple pendulum. The fact that we can get a result with $L_1$ or with $L_2$ simply stems from the fact that $\omega$ is symmetric in the two lengths:it doesn't really matter where the heavier mass is. We would obviously choose $L_1$ in our case, because the other one makes no sense.
\end{inparaenum}
\end{solution}

\question Two equal masses are constrained to move without friction, one on the positive $x$ axis and one on the positive $y$ axis. They are attached to two identical springs (force constant $k$) whose other ends are attached to the origin. In addition, the two masses are connected to each other by a third spring of force constant $k'$. The springs are chosen so that the system is in equilibrium with all three springs relaxed. What are the normal frequencies? Find and describe the normal modes.

\begin{solution}
	As usual, the complicated part about this problem is figuring out the potential energy. The issue here is the annoying $k'$ spring, which is diagonal. Now, drawing springs is really hard, so just pay attention to my description.
	
	Let $L$ be the length of the $k'$ spring when the other two are stretched a distance $x$ and $y$ respectively, and let $\ell$ be the rest length of the horizontal springs. From basic trigonometry, the rest length of the $k'$ spring is $\sqrt{2}\ell$. Now, the distance we're actually interested in in $\Delta L=L-\sqrt{2}\ell$, since this is what determines the force this spring exerts. In what follows, I'll assume $x$ and $y$ are small, so I'll only keep terms up to second order in them.
	
	We can express $\Delta L$ as:
	\begin{eqnarray*}
		\Delta L&=&\sqrt{(\ell+x)^2+(\ell+y)^2}-\sqrt{2}\ell\\
		&=& \sqrt{2\ell^2+x^2+y^2+2\ell x+2\ell y}-\sqrt{2}\ell\\
		&=&\sqrt{2}\ell\sqrt{1+\frac{x^2+y^2}{2\ell^2}+\frac{x+y}{\ell}}-\sqrt{2}\ell\\
		&\approx& \sqrt{2}\ell\left(1+\frac{x^2+y^2}{4\ell^2}+\frac{x+y}{2\ell}-1\right)\\
		\Delta L&=&\frac{\sqrt{2}}{2}\left(\frac{x^2+y^2}{2\ell}+x+y\right)
	\end{eqnarray*}
	
	With that, our potential energy is
	\begin{eqnarray*}
		U&=&=\frac{1}{2}k(x^2+y^2)+\frac{1}{2}k'\Delta L^2\\
		&=&\frac{1}{2}k(x^2+y^2)+\frac{1}{4}k'\left(\frac{x^2+y^2}{2\ell}+x+y\right)^2\\
		&\approx& \frac{1}{2}k(x^2+y^2)+\frac{1}{4}k'\left(x^2+y^2+2xy\right)
	\end{eqnarray*}
	where in the last step I got rid of all terms of order higher than 2.
	
	With that potential energy, our Lagrangian becomes $$\mathcal{L}=\frac{1}{2}m(\dot{x}^2+\dot{y}^2)-\frac{1}{2}k(x^2+y^2)-\frac{1}{4}k'\left(x^2+y^2+2xy\right)$$ so our equations of motion are
	\begin{eqnarray*}
		m\ddot{x}&=&-x\left(k+\frac{1}{2}k'\right)-\frac{1}{2}k'y\\
		m\ddot{y}&=&-y\left(k+\frac{1}{2}k'\right)-\frac{1}{2}k'x
	\end{eqnarray*}
	or $\mathbf{M\ddot{x}}=-\mathbf{Kx}$, where
	\[\mathbf{M}=
	\begin{pmatrix} m&0\\0&m \end{pmatrix}
	,\hspace{0.5cm}\text{and}\hspace{0.5cm} \mathbf{K}=
	\begin{pmatrix}
		k+\frac{1}{2}k'&\frac{1}{2}k'\\
		\frac{1}{2}k'&k+\frac{1}{2}k'
	\end{pmatrix}\]
	
	Now we do our usual thing: assume oscillatory solutions and re-write the matrix equation in terms of that, which will lead us to require that det$(\mathbf{K}-\omega^2\mathbf{M})=0$:
	\[\text{det}
	\begin{pmatrix}
		k+\frac{k'}{2}-\omega^2 m&\frac{k'}{2}\\
		\frac{k'}{2}&k+\frac{k'}{2}-\omega^2 m
	\end{pmatrix}=
	\left(k+\frac{k'}{2}-\omega^2m\right)^2-\frac{k'^2}{4}=0\]
	
	Working on the equation:
	$$k^2+kk'+\omega^4m^2-2\omega^2m\left(k+\frac{k'}{2}\right)=0$$
	which gives us
	$$\omega_1=\pm\sqrt{\frac{k}{m}}, \hspace{1cm} \omega_2=\pm\sqrt{\frac{k+k'}{m}}$$
	
	This is already giving us a hint about the normal modes. In the first one, the diagonal spring plays no role, so we expect it isn't stretched. Let's see...
	
	Using $\omega_1$ first, our eigenvector equation becomes:
	\[\mathbf{K}-\omega^2\mathbf{M}=\frac{k'}{2}
	\begin{pmatrix}
		1&1\\
		1&1
	\end{pmatrix} \cdot \begin{pmatrix} a_1\\a_2\end{pmatrix}=\frac{k'}{2}
	\begin{pmatrix}
		a_1+a_2\\
		a_1+a_2
	\end{pmatrix}=0\]
	This means that in this mode $a_1=-a_2$. That is, the horizontal springs are oscillating exactly out of phase and with the same amplitude. As we expected, this means that the diagonal spring is always kept at the same length, so it plays no role in the equations of motion.
	
	Using $\omega_2$ we get:
	\[\mathbf{K}-\omega^2\mathbf{M}=\frac{k'}{2}
	\begin{pmatrix}
		-1&1\\
		1&-1
	\end{pmatrix}\cdot\begin{pmatrix}a_1\\a_2\end{pmatrix}=\frac{k'}{2}
	\begin{pmatrix}
		-a_1+a_2\\
		a_1-a_2
	\end{pmatrix}=0\]
	This means that $a_1=a_2$. In this case, the springs are oscillating perfectly in phase and with the same amplitude, thus stretching and compressing the diagonal spring, so its spring constant plays a role in the frequency of oscillations.


\end{solution}

\question A bead of mass $m$ is threaded on a frictionless circular wire hoop of radius $R$ and mass $m$ (same mass). The hoop is suspended at the point $A$ and it's free to swing in its own vertical plane. Using the angles $\phi_1$ and $\phi_2$ as generalized coordinates, solve for the normal frequencies of small oscillations, and find and describe the motion in the corresponding normal modes.

\begin{solution}
	I actually find Taylor's suggestions for angles unnecessarily complicated, so I'm using a slightly modified version described in the figure. The only difference is that the second angle is with respect to the vertical, not the diameter.
	
	\begin{figure}[h]
		\centering
		\includegraphics[width=0.3\textwidth]{problem3}
		\caption{The variables used in Problem 3}
		\label{prob3fig}
	\end{figure}
	
	Now, with those angles, the coordinates of the mass $m$ are given by:
	\begin{eqnarray*}
		x_m&=&R(\sin\phi_2+\sin\phi_1)\\
		y_m&=&-R(\cos\phi_2+\cos\phi_1)
	\end{eqnarray*}
	and its kinetic energy is given by
	\begin{eqnarray*}
		T_m&=&\frac{1}{2}m(\dot{x}_m^2+\dot{y}_m^2)\\
		&=& \frac{1}{2}mR^2\left[(\dot{\phi}_1\cos\phi_1+\dot{\phi}_2\cos\phi_2)^2+(\dot{\phi}_1\sin\phi_1+\dot{\phi}_2\sin\phi_2)^2\right]\\
		&=&\frac{1}{2}mR^2\left[\dot{\phi}_1^2+\dot{\phi}_2^2+2\dot{\phi}_1\dot{\phi}_2\cos(\phi_1-\phi_2)\right]\\
		&\approx&\frac{1}{2}mR^2(\dot{\phi}_1^2+\dot{\phi}_2^2+2\dot{\phi}_1\dot{\phi}_2)
	\end{eqnarray*}
	where in the last line I used the small angle approximation up to second order.
	
	The kinetic energy of the hoop is only rotational. The only "tricky" part is finding the right moment of inertia, but that's easy, because this is a 2D problem, so $I$ is basically a scalar. We just have to move it form the center to point $A$ by using the parallel axes theorem: $I=I_{\text{cm}}+mR^2=mR^2+mR^2=2mR^2$, so now our total kinetic energy is:
	$$T_{\text{tot}}=mR^2\dot{\phi}_1^2+\frac{1}{2}mR^2(\dot{\phi}_1^2+\dot{\phi}_2^2+2\dot{\phi}_1\dot{\phi}_2)$$
	
	To find the potential energy, all we need is the $y$ coordinate of the center of the hoop. This is given by $y_{\text{hoop}}=-R\cos\phi_1$, so our potential energy is $$U_\text{tot}=-2mgR\cos\phi_1-mgR\cos\phi_2\approx \frac{1}{2}mgR(2\phi_1^2+\phi_2^2)+U_0$$
	where, again, I approximated up to second order and put all the constants in $U_0$ since they won't show up in the equations of motion.
	
	After all the algebra with the Lagrangian, our equations of motion turn out to be:
	\begin{eqnarray*}
		3\ddot{\phi}_1+\ddot{\phi}_2&=&-\frac{2g}{R}\phi_1\\
		\ddot{\phi}_1+\ddot{\phi}_2&=&-\frac{g}{R}\phi_2
	\end{eqnarray*}
	which we can express in matrix notation by defining
	\[\mathbf{M}=
	\begin{pmatrix}
		3&1\\
		1&1
	\end{pmatrix},
	\hspace{1cm}
	\mathbf{K}= \frac{g}{R} \begin{pmatrix}
		2&0\\
		0&1
	\end{pmatrix}\]
	%\vspace{6cm}
	so now our equation is simply $\mathbf{M}\ddot{\boldsymbol{\phi}}=-\mathbf{K}\boldsymbol{\phi}$. The rest is the same old spiel: guess an exponential solution with frequency $\omega$, plug it back in, write in a form that necessitates a zero determinant, solve for $\omega$. So our equation for $\omega$ is lkjahsdkjhwkegkdfaksdkagsdjhgajhsdgajkgsdkjahskdjhakjsdhkaj aksjdhkajhsdkjha aksjdhkahjsdk asdkjhaksjda skjx kajshdkjbasc e:
	\end{solution}
\end{questions}
\end{document}
	
yaelcita
Posts: 6
Joined: Sat May 29, 2010 11:46 pm

My page breaks are being ignored!

Post by yaelcita »

Also, in case this happens to me only, here's a picture of it. Scroll to the bottom. Notice how there's writing over the page number, even.
The page where the problem is
The page where the problem is
Screen shot 2010-05-30 at 3.30.46 PM.png (154.23 KiB) Viewed 6288 times
User avatar
localghost
Site Moderator
Posts: 9202
Joined: Fri Feb 02, 2007 12:06 pm

My page breaks are being ignored!

Post by localghost »

Your example works fine for me as is. I only had to set the demo option for the graphicx package because the external graphics file is not present to me.

Code: Select all

 *File List*
    exam.cls    2008/07/19 Version 2.3 by Philip Hirschhorn
  ifthen.sty    2001/05/26 v1.1c Standard LaTeX ifthen package (DPC)
 article.cls    2007/10/19 v1.4h Standard LaTeX document class
  size11.clo    2007/10/19 v1.4h Standard LaTeX file (size option)
graphicx.sty    1999/02/16 v1.0f Enhanced LaTeX Graphics (DPC,SPQR)
  keyval.sty    1999/03/16 v1.13 key=value parser (DPC)
graphics.sty    2009/02/05 v1.0o Standard LaTeX Graphics (DPC,SPQR)
    trig.sty    1999/03/16 v1.09 sin cos tan (DPC)
graphics.cfg    2009/08/28 v1.8 graphics configuration of TeX Live
  pdftex.def    2009/08/25 v0.04m Graphics/color for pdfTeX
 amsmath.sty    2000/07/18 v2.13 AMS math features
 amstext.sty    2000/06/29 v2.01
  amsgen.sty    1999/11/30 v2.0
  amsbsy.sty    1999/11/29 v1.2d
  amsopn.sty    1999/12/14 v2.01 operator names
 amssymb.sty    2009/06/22 v3.00
amsfonts.sty    2009/06/22 v3.00 Basic AMSFonts support
enumerate.sty    1999/03/05 v3.00 enumerate extensions (DPC)
paralist.sty    2002/03/18 v2.3b Extended list environments (BS)
verbatim.sty    2003/08/22 v1.5q LaTeX2e package for verbatim enhancements
geometry.sty    2010/03/13 v5.3 Page Geometry
   ifpdf.sty    2009/04/10 v2.0 Provides the ifpdf switch (HO)
  ifvtex.sty    2008/11/04 v1.4 Switches for detecting VTeX and its modes (HO)
geometry.cfg
supp-pdf.mkii
    umsa.fd    2009/06/22 v3.00 AMS symbols A
    umsb.fd    2009/06/22 v3.00 AMS symbols B
 ***********
Looking at your file list it seems that you have an older LaTeX system (e. g. take a look at the version of the »article« class). This can affect the final output. The result on my Linux system² is attached.

Furthermore you should not use the eqnarray environment [1]. It generates inconsistencies and can cause other problems.

As soon as the problem is soirted out, please act according to Section 3 of the Board Rules (last two paragraphs).

[1] The PracTeX Journal - TeX Users Group, Lars Madsen: Avoid eqnarray!


Best regards and welcome to the board
Thorsten
Attachments
temp.pdf
Final output of the given example.
(140.1 KiB) Downloaded 255 times
meho_r
Posts: 823
Joined: Tue Aug 07, 2007 5:28 pm

My page breaks are being ignored!

Post by meho_r »

No problem here, TeXLive2009, on Ubuntu Linux. What to do next?

Firstly, make correction suggested by localghost regarding eqnarray environment.

Secondly, update the outdated packages. Here are the packages from the list you provided that are older than those present on my machine:

Code: Select all

*File List*

article.cls    2005/09/16 v1.4f Standard LaTeX document class
size11.clo    2005/09/16 v1.4f Standard LaTeX file (size option)

graphics.sty    2006/02/20 v1.0o Standard LaTeX Graphics (DPC,SPQR)
    
graphics.cfg    2007/01/18 v1.5 graphics configuration of teTeX/TeXLive
pdftex.def    2008/09/08 v0.04l Graphics/color for pdfTeX

amssymb.sty    2002/01/22 v2.2d
amsfonts.sty    2001/10/25 v2.2f

geometry.sty    2008/12/21 v4.2 Page Geometry
ifpdf.sty    2007/12/12 v1.6 Provides the ifpdf switch (HO)
ifvtex.sty    2008/11/04 v1.4 Switches for detecting VTeX and its modes (HO)

umsa.fd    2002/01/19 v2.2g AMS font definitions
umsb.fd    2002/01/19 v2.2g AMS font definitions

***********
I suggest you update these packages one by one and test the output after every update, so that you can learn which one was causing the trouble (and avoid potential new troubles). Do the backup first, of course. And, don't forget to run texhash after every replacement.

There is a way to locally install new, updated packages, without messing with those already installed (e.g. frabjous wrote about this). Just as an illustration, on my system, I tested adding a new memoir document class version; I downloaded it, run latex on .ins file to create all necessary files and put them in ~/.texlive2009/texmf-var/tex/latex/memoir (.texlive2009 directory was automatically created during TL2009 installation and it actually resembles original path in TeX directory structure, marked by red color, i.e., /usr/share/texmf-texlive/tex/latex/memoir). The last step was sudo texhash to update the database. From the screenshot, I assume you're using a Mac, and I'm not sure if things are the same, so you may have to look around.
yaelcita
Posts: 6
Joined: Sat May 29, 2010 11:46 pm

Re: My page breaks are being ignored!

Post by yaelcita »

Thanks everyone! I haven't tried those suggestions yet, cause I'm very busy with other things at the moment, but I'll let you know how it went once I do. I was thinking I probably needed to update something, but wasn't sure what, so this was very helpful.

Also, thanks for the reference about eqnarray. I had read it was bad, but I didn't really know why. The weird spacing doesn't really bother me, but now I see there's other issues too, so I guess I'll stop using it.
yaelcita
Posts: 6
Joined: Sat May 29, 2010 11:46 pm

My page breaks are being ignored!

Post by yaelcita »

Nothing works!!! I just updated all my old packages (I had an old version of TexLive, and the updater was configured wrong, so it's good I took care of that, anyway). I also changed all my \eqnarray to \align, but the problem is still there. Here's my new file list, in case someone can find something wrong with it.

Code: Select all

*File List*
    exam.cls    2008/07/19 Version 2.3 by Philip Hirschhorn
  ifthen.sty    2001/05/26 v1.1c Standard LaTeX ifthen package (DPC)
 article.cls    2007/10/19 v1.4h Standard LaTeX document class
  size11.clo    2007/10/19 v1.4h Standard LaTeX file (size option)
graphicx.sty    1999/02/16 v1.0f Enhanced LaTeX Graphics (DPC,SPQR)
  keyval.sty    1999/03/16 v1.13 key=value parser (DPC)
graphics.sty    2009/02/05 v1.0o Standard LaTeX Graphics (DPC,SPQR)
    trig.sty    1999/03/16 v1.09 sin cos tan (DPC)
graphics.cfg    2009/08/28 v1.8 graphics configuration of TeX Live
  pdftex.def    2010/02/14 v0.04n Graphics/color for pdfTeX
   color.sty    2005/11/14 v1.0j Standard LaTeX Color (DPC)
   color.cfg    2007/01/18 v1.5 color configuration of teTeX/TeXLive
 amsmath.sty    2000/07/18 v2.13 AMS math features
 amstext.sty    2000/06/29 v2.01
  amsgen.sty    1999/11/30 v2.0
  amsbsy.sty    1999/11/29 v1.2d
  amsopn.sty    1999/12/14 v2.01 operator names
 amssymb.sty    2009/06/22 v3.00
amsfonts.sty    2009/06/22 v3.00 Basic AMSFonts support
enumerate.sty    1999/03/05 v3.00 enumerate extensions (DPC)
paralist.sty    2002/03/18 v2.3b Extended list environments (BS)
verbatim.sty    2003/08/22 v1.5q LaTeX2e package for verbatim enhancements
geometry.sty    2010/03/13 v5.3 Page Geometry
   ifpdf.sty    2010/01/28 v2.1 Provides the ifpdf switch (HO)
  ifvtex.sty    2010/03/01 v1.5 Switches for detecting VTeX and its modes (HO)
supp-pdf.mkii
    umsa.fd    2009/06/22 v3.00 AMS symbols A
    umsb.fd    2009/06/22 v3.00 AMS symbols B
problem3.png    Graphic file (type png)
problem4.png    Graphic file (type png)
Also, here's the entire file... As I said before, sometimes the problem shows up only after so many pages, so maybe cutting it short changed it last time. I realize this is not exactly a MWE, but I don't understand what's going on here... This time, it's only page 6 that has an issue.

I'm not entirely sure about this, but going over my files changing eqnarray to align, i think I have finally found something in common among all the problem files. The issue seems to happen after the first figure in the document. I'm not sure it's always like this, cause I haven't checked all files yet, but, could this be the cause of the problem? And if so, what am I doing wrong? I just changed my image file to a jpg, to see if that made a difference, but it didn't.

Code: Select all

\documentclass[a4paper,11pt,answers]{exam}
\usepackage[pdftex]{graphicx}
\printanswers
\qformat{\textbf{Problem \thequestion}\hfill}
\noaddpoints

\renewcommand{\solutiontitle}{\textbf{Solution:}\par\noindent}
\renewenvironment{TheSolution}
  {
    \vspace{\parskip}
      \leftskip=0pt
    \rightskip=0pt
    \solutiontitle
    \ignorespaces
  }

\usepackage{amsmath}
\usepackage{amssymb}
%\usepackage{setspace}
\usepackage{enumerate}
\usepackage{paralist}
\usepackage{verbatim}

\newcommand{\dis}{\displaystyle}
\usepackage[left=2cm,top=2cm,right=3cm,nohead,nofoot]{geometry}
\footskip=1cm

\author{me}

\title{PHYSICS}

\date{}

\begin{document}
\maketitle

\centering{\LARGE{Homework 6 Solutions}}

\begin{questions}

\question \begin{inparaenum} [\bfseries(a)] \item Find the normal frequencies for small oscillations of the double pendulum for arbitrary values of the masses and lengths.

\item Check that your answers are correct for the special case that $m_1=m_2$ and $L_1=L_2$.

\item Discuss the limit that $m_2\rightarrow 0$
\end{inparaenum}

\begin{solution}
	\begin{inparaenum}[\bfseries(a)]

	\item The entire first part of this problem is actually worked out in Taylor as an example (pages 431-434), so I won't re-derive it here. Instead, I'll take over from where Taylor left off.
	
	The equation of motion is $\mathbf{M}\boldsymbol{\ddot{\phi}}=-\mathbf{K}\boldsymbol{\phi}$, where $\mathbf{M}$ and $\mathbf{K}$ are given by equation 11.44 in Taylor:
	\[ \mathbf{M}= 
	\begin{pmatrix}
		(m_1+m_2)L_1^2&m_2L_1L_2\\
		m_2L_1L_2&m_2L_2^2
	\end{pmatrix}, \hspace{1cm}
	\mathbf{K}=
	\begin{pmatrix}
		(m_1+m_2)gL_1&0\\
		0& m_2gL_2
	\end{pmatrix}
	\]
	
	The normal frequencies are found by proposing oscillatory solutions, writing them as the real part of an exponential, plugging them back into the equation of motion, and then solving $(\mathbf{K}-\omega^2\mathbf{M})\mathbf{a}=0$, which only has solutions when det$(\mathbf{K}-\omega^2\mathbf{M})=0$. So we just have to find the values of $\omega$ that satisfy that determinant equation. I did this entirely in Mathematica, because it's ugly and messy. I invite you to do the same. The answer is:
	$$ \omega^2=\frac{g(L_1+L_2)(m_1+m_2)\pm g\sqrt{(m_1+m_2)\left[(L_1-L_2)^2m_1+(L_1+L_2)^2m_2\right]}}{2L_1L_2m_1}$$
	which is just scary.
	
	Notice we can re-write the equation for $\mathbf{a}$ to make it more clear that it's just an eigenvalue equation: 
	\begin{align*}
		(\mathbf{K}-\omega^2\mathbf{M})\mathbf{a}&=0\\
		\mathbf{K\, a}&= \omega^2\mathbf{M \,a}\\
		\mathbf{M}^{-1}\mathbf{K \,a}&=\omega^2\mathbf{a}
	\end{align*}
	so another way to find our values of $\omega$ is to see that they are just the eigenvalues of that matrix.
   
   	\item When $m_1=m_2$ and $L_1=L_2$, the first term in the root cancels, and the rest is:
   	$$ \omega^2=\frac{g2L\cdot2m\pm g\sqrt{2m^2\,4L^2}}{2L^2m}=\frac{g2L\cdot2m\pm g\,2m\cdot L\sqrt{2}}{2L^2m}=\frac{g}{L}\left(2\pm\sqrt{2}\right)$$
	which is exactly equation 11.47 where Taylor describes that example.
	
	\item In the case when $m_2=0$ we get:
	$$\omega^2=\frac{g(L_1+L_2)m_1\pm g\sqrt{m_1^2(L_1-L_2)^2}}{2L_1L_2m_1}=\frac{g\left(L_1+L_2\pm(L_1-L_2)\right)}{2L_1L_2}=\frac{g}{L_{1,2}}$$
   which is what we would expect: the pendulum has been reduced to a simple pendulum. The fact that we can get a result with $L_1$ or with $L_2$ simply stems from the fact that $\omega$ is symmetric in the two lengths:it doesn't really matter where the heavier mass is. We would obviously choose $L_1$ in our case, because the other one makes no sense.
\end{inparaenum}
\end{solution}

\question Two equal masses are constrained to move without friction, one on the positive $x$ axis and one on the positive $y$ axis. They are attached to two identical springs (force constant $k$) whose other ends are attached to the origin. In addition, the two masses are connected to each other by a third spring of force constant $k'$. The springs are chosen so that the system is in equilibrium with all three springs relaxed. What are the normal frequencies? Find and describe the normal modes.

\begin{solution}
	As usual, the complicated part about this problem is figuring out the potential energy. The issue here is the annoying $k'$ spring, which is diagonal. Now, drawing springs is really hard, so just pay attention to my description.
	
	Let $L$ be the length of the $k'$ spring when the other two are stretched a distance $x$ and $y$ respectively, and let $\ell$ be the rest length of the horizontal springs. From basic trigonometry, the rest length of the $k'$ spring is $\sqrt{2}\ell$. Now, the distance we're actually interested in in $\Delta L=L-\sqrt{2}\ell$, since this is what determines the force this spring exerts. In what follows, I'll assume $x$ and $y$ are small, so I'll only keep terms up to second order in them.
	
	We can express $\Delta L$ as:
	\begin{align*}
		\Delta L&=\sqrt{(\ell+x)^2+(\ell+y)^2}-\sqrt{2}\ell\\
		&= \sqrt{2\ell^2+x^2+y^2+2\ell x+2\ell y}-\sqrt{2}\ell\\
		&=\sqrt{2}\ell\sqrt{1+\frac{x^2+y^2}{2\ell^2}+\frac{x+y}{\ell}}-\sqrt{2}\ell\\
		&\approx \sqrt{2}\ell\left(1+\frac{x^2+y^2}{4\ell^2}+\frac{x+y}{2\ell}-1\right)\\
		\Delta L&=\frac{\sqrt{2}}{2}\left(\frac{x^2+y^2}{2\ell}+x+y\right)
	\end{align*}
	
	With that, our potential energy is
	\begin{align*}
		U&==\frac{1}{2}k(x^2+y^2)+\frac{1}{2}k'\Delta L^2\\
		&=\frac{1}{2}k(x^2+y^2)+\frac{1}{4}k'\left(\frac{x^2+y^2}{2\ell}+x+y\right)^2\\
		&\approx \frac{1}{2}k(x^2+y^2)+\frac{1}{4}k'\left(x^2+y^2+2xy\right)
	\end{align*}
	where in the last step I got rid of all terms of order higher than 2.
	
	With that potential energy, our Lagrangian becomes $$\mathcal{L}=\frac{1}{2}m(\dot{x}^2+\dot{y}^2)-\frac{1}{2}k(x^2+y^2)-\frac{1}{4}k'\left(x^2+y^2+2xy\right)$$ so our equations of motion are
	\begin{align*}
		m\ddot{x}&=-x\left(k+\frac{1}{2}k'\right)-\frac{1}{2}k'y\\
		m\ddot{y}&=-y\left(k+\frac{1}{2}k'\right)-\frac{1}{2}k'x
	\end{align*}
	or $\mathbf{M\ddot{x}}=-\mathbf{Kx}$, where
	\[\mathbf{M}=
	\begin{pmatrix} m&0\\0&m \end{pmatrix}
	,\hspace{0.5cm}\text{and}\hspace{0.5cm} \mathbf{K}=
	\begin{pmatrix}
		k+\frac{1}{2}k'&\frac{1}{2}k'\\
		\frac{1}{2}k'&k+\frac{1}{2}k'
	\end{pmatrix}\]
	
	Now we do our usual thing: assume oscillatory solutions and re-write the matrix equation in terms of that, which will lead us to require that det$(\mathbf{K}-\omega^2\mathbf{M})=0$:
	\[\text{det}
	\begin{pmatrix}
		k+\frac{k'}{2}-\omega^2 m&\frac{k'}{2}\\
		\frac{k'}{2}&k+\frac{k'}{2}-\omega^2 m
	\end{pmatrix}=
	\left(k+\frac{k'}{2}-\omega^2m\right)^2-\frac{k'^2}{4}=0\]
	
	Working on the equation:
	$$k^2+kk'+\omega^4m^2-2\omega^2m\left(k+\frac{k'}{2}\right)=0$$
	which gives us
	$$\omega_1=\pm\sqrt{\frac{k}{m}}, \hspace{1cm} \omega_2=\pm\sqrt{\frac{k+k'}{m}}$$
	
	This is already giving us a hint about the normal modes. In the first one, the diagonal spring plays no role, so we expect it isn't stretched. Let's see...
	
	Using $\omega_1$ first, our eigenvector equation becomes:
	\[\mathbf{K}-\omega^2\mathbf{M}=\frac{k'}{2}
	\begin{pmatrix}
		1&1\\
		1&1
	\end{pmatrix} \cdot \begin{pmatrix} a_1\\a_2\end{pmatrix}=\frac{k'}{2}
	\begin{pmatrix}
		a_1+a_2\\
		a_1+a_2
	\end{pmatrix}=0\]
	This means that in this mode $a_1=-a_2$. That is, the horizontal springs are oscillating exactly out of phase and with the same amplitude. As we expected, this means that the diagonal spring is always kept at the same length, so it plays no role in the equations of motion.
	
	Using $\omega_2$ we get:
	\[\mathbf{K}-\omega^2\mathbf{M}=\frac{k'}{2}
	\begin{pmatrix}
		-1&1\\
		1&-1
	\end{pmatrix}\cdot\begin{pmatrix}a_1\\a_2\end{pmatrix}=\frac{k'}{2}
	\begin{pmatrix}
		-a_1+a_2\\
		a_1-a_2
	\end{pmatrix}=0\]
	This means that $a_1=a_2$. In this case, the springs are oscillating perfectly in phase and with the same amplitude, thus stretching and compressing the diagonal spring, so its spring constant plays a role in the frequency of oscillations.


\end{solution}

\question A bead of mass $m$ is threaded on a frictionless circular wire hoop of radius $R$ and mass $m$ (same mass). The hoop is suspended at the point $A$ and it's free to swing in its own vertical plane. Using the angles $\phi_1$ and $\phi_2$ as generalized coordinates, solve for the normal frequencies of small oscillations, and find and describe the motion in the corresponding normal modes.

\begin{solution}
	I actually find Taylor's suggestions for angles unnecessarily complicated, so I'm using a slightly modified version described in the figure. The only difference is that the second angle is with respect to the vertical, not the diameter.
	
	\begin{figure}[h]
		\centering
		\includegraphics[width=0.3\textwidth]{problem3}
		\caption{The variables used in Problem 3}
		\label{prob3fig}
	\end{figure}
	
	Now, with those angles, the coordinates of the mass $m$ are given by:
	\begin{align*}
		x_m&=R(\sin\phi_2+\sin\phi_1)\\
		y_m&=-R(\cos\phi_2+\cos\phi_1)
	\end{align*}
	and its kinetic energy is given by
	\begin{align*}
		T_m&=\frac{1}{2}m(\dot{x}_m^2+\dot{y}_m^2)\\
		&= \frac{1}{2}mR^2\left[(\dot{\phi}_1\cos\phi_1+\dot{\phi}_2\cos\phi_2)^2+(\dot{\phi}_1\sin\phi_1+\dot{\phi}_2\sin\phi_2)^2\right]\\
		&=\frac{1}{2}mR^2\left[\dot{\phi}_1^2+\dot{\phi}_2^2+2\dot{\phi}_1\dot{\phi}_2\cos(\phi_1-\phi_2)\right]\\
		&\approx\frac{1}{2}mR^2(\dot{\phi}_1^2+\dot{\phi}_2^2+2\dot{\phi}_1\dot{\phi}_2)
	\end{align*}
	where in the last line I used the small angle approximation up to second order.
	
	The kinetic energy of the hoop is only rotational. The only "tricky" part is finding the right moment of inertia, but that's easy, because this is a 2D problem, so $I$ is basically a scalar. We just have to move it form the center to point $A$ by using the parallel axes theorem: $I=I_{\text{cm}}+mR^2=mR^2+mR^2=2mR^2$, so now our total kinetic energy is:
	$$T_{\text{tot}}=mR^2\dot{\phi}_1^2+\frac{1}{2}mR^2(\dot{\phi}_1^2+\dot{\phi}_2^2+2\dot{\phi}_1\dot{\phi}_2)$$
	
	To find the potential energy, all we need is the $y$ coordinate of the center of the hoop. This is given by $y_{\text{hoop}}=-R\cos\phi_1$, so our potential energy is $$U_\text{tot}=-2mgR\cos\phi_1-mgR\cos\phi_2\approx \frac{1}{2}mgR(2\phi_1^2+\phi_2^2)+U_0$$
	where, again, I approximated up to second order and put all the constants in $U_0$ since they won't show up in the equations of motion.
	
	After all the algebra with the Lagrangian, our equations of motion turn out to be:
	\begin{align*}
		3\ddot{\phi}_1+\ddot{\phi}_2&=-\frac{2g}{R}\phi_1\\
		\ddot{\phi}_1+\ddot{\phi}_2&=-\frac{g}{R}\phi_2
	\end{align*}
	which we can express in matrix notation by defining
	\[\mathbf{M}=
	\begin{pmatrix}
		3&1\\
		1&1
	\end{pmatrix},
	\hspace{1cm}
	\mathbf{K}= \frac{g}{R} \begin{pmatrix}
		2&0\\
		0&1
	\end{pmatrix}\]
	%\vspace{6cm}
	so now our equation is simply $\mathbf{M}\ddot{\boldsymbol{\phi}}=-\mathbf{K}\boldsymbol{\phi}$. The rest is the same old spiel: guess an exponential solution with frequency $\omega$, plug it back in, write in a form that necessitates a zero determinant, solve for $\omega$. So our equation for $\omega$ is:
	\[\text{det}\begin{pmatrix}
		2\frac{g}{R}-3\omega^2&-\omega^2\\
		-\omega^2&\frac{g}{R}-\omega^2
	\end{pmatrix}=(2\frac{g}{R}-3\omega^2)(\frac{g}{R}-\omega^2)-\omega^4=0\]
	This gives us values of $\omega$:
	$$\omega_1^2=\frac{2g}{R}, \hspace{1cm}\omega_2^2=\frac{g}{2R}$$
	which turns our $\mathbf{a}$ equation into, for $\omega_1$
	\[\begin{pmatrix}
		2\frac{g}{R}-6\frac{g}{R}&-2\frac{g}{R}\\
		-2\frac{g}{R}&\frac{g}{R}-2\frac{g}{R}
	\end{pmatrix}\cdot
	\begin{pmatrix} a_1\\a_2 \end{pmatrix}=-\frac{g}{R}
	\begin{pmatrix}
		4&2\\
		2&1
	\end{pmatrix}\cdot\begin{pmatrix}a_1\\a_2\end{pmatrix}=0\Rightarrow\mathbf{a}=
	\begin{pmatrix}1\\-2\end{pmatrix}\]
	and for $\omega_2$:
	
	\[\begin{pmatrix}
		2\frac{g}{R}-\frac{3g}{2R}&-\frac{g}{2R}\\
		-\frac{g}{2R}&\frac{g}{R}-\frac{g}{2R}
	\end{pmatrix}\cdot
	\begin{pmatrix} a_1\\a_2 \end{pmatrix}=\frac{g}{2R}
	\begin{pmatrix}
		1&-1\\
		-1&1
	\end{pmatrix}\cdot\begin{pmatrix}a_1\\a_2\end{pmatrix}=0\Rightarrow\mathbf{a}=
	\begin{pmatrix}1\\1\end{pmatrix}\]
	
	We can interpret the first solution as movements in opposite directions: the bead goes down when the hoop goes up, and viceversa. Also, the movement of the bead is twice as big as the movement of the loop. This makes some intuitive sense, since we imagine it's easier to move a point particle than an extended one.
	
	The other solution gives movements in phase and with the same amplitude. That is to say, the bead doesn't move with respect to the hoop.
	
\end{solution}

\question \begin{inparaenum}[\bfseries(a)] A simple pendulum of mass $M$ and length $L$ is suspended from a cart of mass $m$ that moves freely along a horizontal track. 

\item What are the normal frequencies?

\item Find and describe the corresponding normal modes
\end{inparaenum}

\begin{solution} \begin{inparaenum}[\bfseries(a)]
	
	The coordinates I'll use are shown in the figure.
	\begin{figure}[h]
		\centering
		\includegraphics[width=0.3\textwidth]{problem4}
		\caption{The variables used in problem 4}
		\label{prob4fig}
	\end{figure}
	
	From the figure we have
	\begin{align*}
		x_M=x_m+L\sin\phi &\Rightarrow \dot{x}_M=\dot{x}_m+L\dot{\phi}\cos\phi\\
		y_M=-L\cos\phi&\Rightarrow\dot{y}_M=L\dot{\phi}\sin\phi
	\end{align*}
	and let me call $x_m=x$ because I'll get tired really soon of writing the subscripts. With those relationships, and approximating the trig functions up to second order, the kinetic and potential energies become:
	\begin{align*}
		T&=T_m+T_M=\frac{1}{2}m\dot{x}^2+\frac{1}{2}M(\dot{x}_M^2+\dot{y}_M^2)\\
		&=\frac{1}{2}m\dot{x}^2+\frac{1}{2}M(\dot{x}^2+L^2\dot{\phi}^2+2L\, \dot{x}\dot{\phi}\cos\phi)\\
		&\approx\frac{1}{2}(m+M)\dot{x}^2+\frac{1}{2}M(L^2\dot{\phi}^2+2L\dot{x}\dot{\phi})\\
		U&= Mgy_M\\
		&=-MgL\,\cos\phi\\
		&\approx -MgL+\frac{1}{2}MgL\,\phi^2
	\end{align*}
	
	After calculating the Lagrange equations of motion we end up with:
	\begin{align*}
		(M+m)\,\ddot{x}+ML\ddot{\phi}&=0\\
		ML\,\ddot{x}+ML^2\ddot{\phi}&=-MgL\phi
	\end{align*}
	or, defining
	\[\mathbf{M}=\begin{pmatrix}
		M+m&ML\\
		ML&ML^2
	\end{pmatrix}, \hspace{1cm} \mathbf{K}=
	\begin{pmatrix}
		0&0\\
		0&MgL
	\end{pmatrix}\]
	we have $\mathbf{M\,\ddot{x}}=-\mathbf{K\,x}$. Now we can calculate all that stuff.
	
	\item The frequencies $\omega$ are, as usual, the eigenvalues of $\mathbf{M}^{-1}\mathbf{K}$, or the solutions of det$(\mathbf{K}-\omega^2\mathbf{M})=0$, which is the same. So:
	\[\text{det}
	\begin{pmatrix}
		-\omega^2(M+m)&-\omega^2ML\\
		-\omega^2ML&MgL-\omega^2ML^2
	\end{pmatrix}=-\omega^2ML(M+m)(g-\omega^2L)-\omega^4M^2L^2=0\]
	Working on that equation:
	\begin{align*}
		-\omega^2ML\left[g(M+m)-\omega^2L(M+m)+\omega^2ML\right]&=0\\
		-\omega^2 ML\left[g(M+m)-\omega^2Lm\right]&=0
	\end{align*}
	so our two normal frequencies are
	$$\omega_1=0\hspace{1cm}\omega_2=\sqrt{\frac{g(M+m)}{Lm}}$$
	
	\item To find the normal modes, we go back to $\mathbf{K}-\omega^2\mathbf{M}=0$ and plug in the values of $\omega$. The first one is easiest:
	\[\begin{pmatrix} 0&0\\0&MgL \end{pmatrix}\cdot
	\begin{pmatrix} a_1\\a_2 \end{pmatrix}=
	\begin{pmatrix} 0\\ a_2 \,MgL\end{pmatrix} =0\hspace{0.5cm}\Rightarrow\hspace{0.5cm} a_2=0\]
	
	Notice this doesn't tell us anything about $a_1$. However, $a_2$ represents $\phi$ in our case, so we can go back to our equations of motion and see what we get when $\phi(t)$ is identically zero. We can see this implies that $\ddot{x}=0$. That means that the cart is moving without acceleration, at a constant velocity. The pendulum doesn't move at all in this case, like we would expect, since it experiences no forces. We would need initial conditions to know more about the movement, but this is certainly a lot coming from a simple zero!
	
	Using our other value for $\omega$ we find:
	\[\begin{pmatrix}
		-g\frac{(M+m)^2}{Lm} & -g(M+m)\frac{M}{m}\\
		-g(M+m)\frac{M}{m} & MgL-gL(M+m)\frac{M}{m}
	\end{pmatrix}\cdot
	\begin{pmatrix}a_1\\a_2\end{pmatrix}=0\]
	So we get an equation that relates $a_1$ to $a_2$:
		$$(M+m)a_1+MLa_2=0\hspace{0.5cm}\Rightarrow\hspace{0.5cm}a_1=-\frac{ML}{M+m}a_2$$
	
	Now, if you recall, this whole $a$ business came about by assuming oscillatory motion, so the fact we get a solution means there is, in fact, oscillatory motion. The frequency of the motion is given by $\omega_2$. As we would expect, this is the frequency of a simple pendulum modified by the two masses. Note that if $m\rightarrow\infty$ then the $M$ in the numerator of $\omega_2$ doesn't matter and we can cancel the $m$'s, recovering the simple pendulum frequency. This is to be expected, since an infinitely massive cart wouldn't move.
	
	Also, the amplitudes of the oscillations of the pendulum and the cart are related via the relationship between $a_1$ and $a_2$. Notice first of all that they are oscillating out of phase, due to the minus sign. Also, $a_1\rightarrow0$ when $m\rightarrow\infty$, as we would expect. 
	
\end{inparaenum}	
\end{solution}

\question \begin{inparaenum}[\bfseries(a)] As a model of a linear triatomic molecule such as CO$_2$, consider a system with two identical atoms each of mass $m$ connected by two identical springs to a single atom of mass $M$. To simplify matters, assume that the system is confined to move in one dimension.

\item Write down the Lagrangian and find the normal frequencies of the system. Show that one of the normal frequencies is zero.

\item Find and describe the motion in the normal modes whose frequencies are nonzero.

\item Do the same for the mode with zero frequency.
\end{inparaenum}

\begin{solution}
\begin{inparaenum}[\bfseries(a)]
	
	\item Since the system can only move in one dimension, we'll have just one coordinate for each atom. Call them $x_1$, $x_2$ and $x_3$. Now, the kinetic energy is easy enough: $T=\frac{1}{2}(m\dot{x}_1^2+m\dot{x}_3^2+M\dot{x}_2^2)$, but the potential energy is slightly more complicated, because the springs stretch and compress due to the different molecules. But it's not so bad. Basically, motion of the middle molecule to the right will compress the right spring and stretch the left one, so we have to add $x_2$ to one of the coordinates and subtract it from the other one. It doesn't really matter which. I've chosen to make it so that $U=\frac{1}{2}k[(x_2-x_1)^2+(x_3-x_2)^2]$
	
	With that, the Lagrangian becomes $$\mathcal{L}=\frac{1}{2}\left[m(\dot{x}_1^2+\dot{x}_3^2)+M\dot{x}_2^2\right]-\frac{1}{2}k\left[(x_2-x_1)^2+(x_3-x_2)^2\right]$$ and our equations of motion are:
	\begin{align*}
		m\ddot{x}_1&=k(x_2-x_1)\\
		M\ddot{x}_2&=k(x_3+x_1)-2kx_2\\
		m\ddot{x}_3&=-k(x_3-x_2)
	\end{align*}
	which leads us to define
	\[\mathbf{M}=
	\begin{pmatrix}
		m&0&0\\
		0&M&0\\
		0&0&m
	\end{pmatrix}, \hspace{1cm}
	\mathbf{K}=k \begin{pmatrix}
		1&-1&0\\
		-1&2&-1\\
		0&-1&1
	\end{pmatrix}\]
	Thus, we now have the equation:
	\[\text{det}\begin{pmatrix}
		k-\omega^2m&-k&0\\
		-k&2k-\omega^2M&-k\\
		0&-k&k-\omega^2m
	\end{pmatrix}= (k-\omega^2m)[(2k-\omega^2M)(k-\omega^2m)-k^2]-k^2(k-\omega^2m)=0\]
	which we can re-write as $$\omega^2\left[-\omega^4m^2M+2km(M+m)\omega^2-k^2(2m+M)\right]=0$$ so we can see that we do get a zero frequency, as advertised. Now we can also get the other two frequencies. Solving the equation with Mathematica we get: 
	\begin{align*}
		\omega_1&=\pm\sqrt{\frac{k}{m}}\\
		\omega_2&=\pm\sqrt{k\left(\frac{1}{m}+\frac{2}{M}\right)}
	\end{align*}
	
	\item Now we want to find the values of $\mathbf{a}$. Our equations are, for the first $\omega_1$:
	\[\begin{pmatrix}
		0&-k&0\\
		-k&2k-kM/m&-k\\
		0&-k&0
	\end{pmatrix}\cdot\begin{pmatrix}a_1\\a_2\\a_3\end{pmatrix}=k
	\begin{pmatrix}
		-a_2\\
		-a_1+(2-M/m)a_2-a_3\\
		-a_2
	\end{pmatrix}=0\]
	So we find that $a_2=0$ and $a_1=-a_3$. This means that the carbon molecule is not moving and the two oxygen molecules are moving in opposite directions. The center of mass of the molecule is not moving, like we would expect.
	
	For $\omega_2$ we get:
	\[\begin{pmatrix}
		-2km/M&-k&0\\
		-k&-k/m&-k\\
		0&-k&-2km/M
	\end{pmatrix}\cdot\begin{pmatrix}a_1\\a_2\\a_3\end{pmatrix}=-k
	\begin{pmatrix}
		2a_1\,m/M+a_2\\
		a_1+a_2/m+a_3\\
		a_2+2a_3m/M
	\end{pmatrix}=0\]
	which means that $a_1=a_3$ and $a_2=-2a_1\,m/M$. That is, in this case, the two oxygens are moving in phase and with the same amplitudes, and the carbon is moving opposite them and with an amplitude determined by the ratio of the masses.
	
	\item Using $\omega=0$ our equation is simply
	\[k\begin{pmatrix}
		1&-1&0\\
		-1&2&-1\\
		0&-1&1
	\end{pmatrix}\cdot\begin{pmatrix}a_1\\a_2\\a_3\end{pmatrix}=k
	\begin{pmatrix}
		a_1-a_2\\
		-a_1+2a_2-a_3\\
		-a_2+a_3
	\end{pmatrix}=0\]
	which means that $a_1=a_2=a_3$. This, together with the fact that the frequency of the motion is zero, tells us that this describes a displacement of the entire molecule rather than a vibration of it.
	
	
\end{inparaenum}
\end{solution}

\end{questions}
\end{document}
I was hoping so much for this to be an outdated packages issue....
torbjorn t.
Posts: 162
Joined: Wed Jun 17, 2009 10:18 pm

My page breaks are being ignored!

Post by torbjorn t. »

The problem also occured for me, in Vista with MikTeX 2.8, but I did find a possible solution, after some trial and error.

One of the things I tried, was to remove your \renewenvironment{TheSolution}, just to see what effect this had. That gave a "not in outer par mode"-error when compiled, so I thought I'd try removing the floats, which seems to have done the trick. At least I didn't experience the same behaviour without the floats.

I replaced the figure-environments with center-environments, and use the caption package to get a caption for the figures. See if the code below works for you as well.

As for why this happens exactly, I can't tell.


Another thing: Please note that

Code: Select all

$$ ... $$
should not be used for display style math. Use

Code: Select all

\[ ... \]
in stead. Refer to l2tabu for details.

Code: Select all

\documentclass[a4paper,11pt,answers]{exam}
\usepackage{graphicx}
\printanswers
\qformat{\textbf{Problem \thequestion}\hfill}
\noaddpoints

\renewcommand{\solutiontitle}{\textbf{Solution:}\par\noindent}
\renewenvironment{TheSolution}
  {
    \vspace{\parskip}
      \leftskip=0pt
    \rightskip=0pt
    \solutiontitle
    \ignorespaces
  }

\usepackage{amsmath}
\usepackage{amssymb}
%\usepackage{setspace}
\usepackage{enumerate}
\usepackage{paralist}
\usepackage{verbatim}
\usepackage{caption}

\newcommand{\dis}{\displaystyle}
\usepackage[left=2cm,top=2cm,right=3cm,nohead,nofoot]{geometry}
\footskip=1cm

\author{me}

\title{PHYSICS}

\date{}

\begin{document}
\maketitle

\centering{\LARGE{Homework 6 Solutions}}

\begin{questions}

\question \begin{inparaenum} [\bfseries(a)] \item Find the normal frequencies for small oscillations of the double pendulum for arbitrary values of the masses and lengths.

\item Check that your answers are correct for the special case that $m_1=m_2$ and $L_1=L_2$.

\item Discuss the limit that $m_2\rightarrow 0$
\end{inparaenum}

\begin{solution}
   \begin{inparaenum}[\bfseries(a)]

   \item The entire first part of this problem is actually worked out in Taylor as an example (pages 431-434), so I won't re-derive it here. Instead, I'll take over from where Taylor left off.
   
   The equation of motion is $\mathbf{M}\boldsymbol{\ddot{\phi}}=-\mathbf{K}\boldsymbol{\phi}$, where $\mathbf{M}$ and $\mathbf{K}$ are given by equation 11.44 in Taylor:
   \[ \mathbf{M}= 
   \begin{pmatrix}
      (m_1+m_2)L_1^2&m_2L_1L_2\\
      m_2L_1L_2&m_2L_2^2
   \end{pmatrix}, \hspace{1cm}
   \mathbf{K}=
   \begin{pmatrix}
      (m_1+m_2)gL_1&0\\
      0& m_2gL_2
   \end{pmatrix}
   \]
   
   The normal frequencies are found by proposing oscillatory solutions, writing them as the real part of an exponential, plugging them back into the equation of motion, and then solving $(\mathbf{K}-\omega^2\mathbf{M})\mathbf{a}=0$, which only has solutions when det$(\mathbf{K}-\omega^2\mathbf{M})=0$. So we just have to find the values of $\omega$ that satisfy that determinant equation. I did this entirely in Mathematica, because it's ugly and messy. I invite you to do the same. The answer is:
   \[ \omega^2=\frac{g(L_1+L_2)(m_1+m_2)\pm g\sqrt{(m_1+m_2)\left[(L_1-L_2)^2m_1+(L_1+L_2)^2m_2\right]}}{2L_1L_2m_1}\]
   which is just scary.
   
   Notice we can re-write the equation for $\mathbf{a}$ to make it more clear that it's just an eigenvalue equation: 
   \begin{align*}
      (\mathbf{K}-\omega^2\mathbf{M})\mathbf{a}&=0\\
      \mathbf{K\, a}&= \omega^2\mathbf{M \,a}\\
      \mathbf{M}^{-1}\mathbf{K \,a}&=\omega^2\mathbf{a}
   \end{align*}
   so another way to find our values of $\omega$ is to see that they are just the eigenvalues of that matrix.
   
      \item When $m_1=m_2$ and $L_1=L_2$, the first term in the root cancels, and the rest is:
      \[ \omega^2=\frac{g2L\cdot2m\pm g\sqrt{2m^2\,4L^2}}{2L^2m}=\frac{g2L\cdot2m\pm g\,2m\cdot L\sqrt{2}}{2L^2m}=\frac{g}{L}\left(2\pm\sqrt{2}\right)\]
   which is exactly equation 11.47 where Taylor describes that example.
   
   \item In the case when $m_2=0$ we get:
   \[\omega^2=\frac{g(L_1+L_2)m_1\pm g\sqrt{m_1^2(L_1-L_2)^2}}{2L_1L_2m_1}=\frac{g\left(L_1+L_2\pm(L_1-L_2)\right)}{2L_1L_2}=\frac{g}{L_{1,2}}\]
   which is what we would expect: the pendulum has been reduced to a simple pendulum. The fact that we can get a result with $L_1$ or with $L_2$ simply stems from the fact that $\omega$ is symmetric in the two lengths:it doesn't really matter where the heavier mass is. We would obviously choose $L_1$ in our case, because the other one makes no sense.
\end{inparaenum}
\end{solution}

\question Two equal masses are constrained to move without friction, one on the positive $x$ axis and one on the positive $y$ axis. They are attached to two identical springs (force constant $k$) whose other ends are attached to the origin. In addition, the two masses are connected to each other by a third spring of force constant $k'$. The springs are chosen so that the system is in equilibrium with all three springs relaxed. What are the normal frequencies? Find and describe the normal modes.

\begin{solution}
   As usual, the complicated part about this problem is figuring out the potential energy. The issue here is the annoying $k'$ spring, which is diagonal. Now, drawing springs is really hard, so just pay attention to my description.
   
   Let $L$ be the length of the $k'$ spring when the other two are stretched a distance $x$ and $y$ respectively, and let $\ell$ be the rest length of the horizontal springs. From basic trigonometry, the rest length of the $k'$ spring is $\sqrt{2}\ell$. Now, the distance we're actually interested in in $\Delta L=L-\sqrt{2}\ell$, since this is what determines the force this spring exerts. In what follows, I'll assume $x$ and $y$ are small, so I'll only keep terms up to second order in them.
   
   We can express $\Delta L$ as:
   \begin{align*}
      \Delta L&=\sqrt{(\ell+x)^2+(\ell+y)^2}-\sqrt{2}\ell\\
      &= \sqrt{2\ell^2+x^2+y^2+2\ell x+2\ell y}-\sqrt{2}\ell\\
      &=\sqrt{2}\ell\sqrt{1+\frac{x^2+y^2}{2\ell^2}+\frac{x+y}{\ell}}-\sqrt{2}\ell\\
      &\approx \sqrt{2}\ell\left(1+\frac{x^2+y^2}{4\ell^2}+\frac{x+y}{2\ell}-1\right)\\
      \Delta L&=\frac{\sqrt{2}}{2}\left(\frac{x^2+y^2}{2\ell}+x+y\right)
   \end{align*}
   
   With that, our potential energy is
   \begin{align*}
      U&==\frac{1}{2}k(x^2+y^2)+\frac{1}{2}k'\Delta L^2\\
      &=\frac{1}{2}k(x^2+y^2)+\frac{1}{4}k'\left(\frac{x^2+y^2}{2\ell}+x+y\right)^2\\
      &\approx \frac{1}{2}k(x^2+y^2)+\frac{1}{4}k'\left(x^2+y^2+2xy\right)
   \end{align*}
   where in the last step I got rid of all terms of order higher than 2.
   
   With that potential energy, our Lagrangian becomes \[\mathcal{L}=\frac{1}{2}m(\dot{x}^2+\dot{y}^2)-\frac{1}{2}k(x^2+y^2)-\frac{1}{4}k'\left(x^2+y^2+2xy\right)\] so our equations of motion are
   \begin{align*}
      m\ddot{x}&=-x\left(k+\frac{1}{2}k'\right)-\frac{1}{2}k'y\\
      m\ddot{y}&=-y\left(k+\frac{1}{2}k'\right)-\frac{1}{2}k'x
   \end{align*}
   or $\mathbf{M\ddot{x}}=-\mathbf{Kx}$, where
   \[\mathbf{M}=
   \begin{pmatrix} m&0\\0&m \end{pmatrix}
   ,\hspace{0.5cm}\text{and}\hspace{0.5cm} \mathbf{K}=
   \begin{pmatrix}
      k+\frac{1}{2}k'&\frac{1}{2}k'\\
      \frac{1}{2}k'&k+\frac{1}{2}k'
   \end{pmatrix}\]
   
   Now we do our usual thing: assume oscillatory solutions and re-write the matrix equation in terms of that, which will lead us to require that det$(\mathbf{K}-\omega^2\mathbf{M})=0$:
   \[\text{det}
   \begin{pmatrix}
      k+\frac{k'}{2}-\omega^2 m&\frac{k'}{2}\\
      \frac{k'}{2}&k+\frac{k'}{2}-\omega^2 m
   \end{pmatrix}=
   \left(k+\frac{k'}{2}-\omega^2m\right)^2-\frac{k'^2}{4}=0\]
   
   Working on the equation:
   \[k^2+kk'+\omega^4m^2-2\omega^2m\left(k+\frac{k'}{2}\right)=0\]
   which gives us
   \[\omega_1=\pm\sqrt{\frac{k}{m}}, \hspace{1cm} \omega_2=\pm\sqrt{\frac{k+k'}{m}}\]
   
   This is already giving us a hint about the normal modes. In the first one, the diagonal spring plays no role, so we expect it isn't stretched. Let's see...
   
   Using $\omega_1$ first, our eigenvector equation becomes:
   \[\mathbf{K}-\omega^2\mathbf{M}=\frac{k'}{2}
   \begin{pmatrix}
      1&1\\
      1&1
   \end{pmatrix} \cdot \begin{pmatrix} a_1\\a_2\end{pmatrix}=\frac{k'}{2}
   \begin{pmatrix}
      a_1+a_2\\
      a_1+a_2
   \end{pmatrix}=0\]
   This means that in this mode $a_1=-a_2$. That is, the horizontal springs are oscillating exactly out of phase and with the same amplitude. As we expected, this means that the diagonal spring is always kept at the same length, so it plays no role in the equations of motion.
   
   Using $\omega_2$ we get:
   \[\mathbf{K}-\omega^2\mathbf{M}=\frac{k'}{2}
   \begin{pmatrix}
      -1&1\\
      1&-1
   \end{pmatrix}\cdot\begin{pmatrix}a_1\\a_2\end{pmatrix}=\frac{k'}{2}
   \begin{pmatrix}
      -a_1+a_2\\
      a_1-a_2
   \end{pmatrix}=0\]
   This means that $a_1=a_2$. In this case, the springs are oscillating perfectly in phase and with the same amplitude, thus stretching and compressing the diagonal spring, so its spring constant plays a role in the frequency of oscillations.


\end{solution}

\question A bead of mass $m$ is threaded on a frictionless circular wire hoop of radius $R$ and mass $m$ (same mass). The hoop is suspended at the point $A$ and it's free to swing in its own vertical plane. Using the angles $\phi_1$ and $\phi_2$ as generalized coordinates, solve for the normal frequencies of small oscillations, and find and describe the motion in the corresponding normal modes.

\begin{solution}
   I actually find Taylor's suggestions for angles unnecessarily complicated, so I'm using a slightly modified version described in the figure. The only difference is that the second angle is with respect to the vertical, not the diameter.
   
   \begin{center}
      \includegraphics[width=0.3\textwidth]{problem3}
      \captionof{figure}{The variables used in Problem 3}
      \label{prob3fig}
   \end{center}
   
   Now, with those angles, the coordinates of the mass $m$ are given by:
   \begin{align*}
      x_m&=R(\sin\phi_2+\sin\phi_1)\\
      y_m&=-R(\cos\phi_2+\cos\phi_1)
   \end{align*}
   and its kinetic energy is given by
   \begin{align*}
      T_m&=\frac{1}{2}m(\dot{x}_m^2+\dot{y}_m^2)\\
      &= \frac{1}{2}mR^2\left[(\dot{\phi}_1\cos\phi_1+\dot{\phi}_2\cos\phi_2)^2+(\dot{\phi}_1\sin\phi_1+\dot{\phi}_2\sin\phi_2)^2\right]\\
      &=\frac{1}{2}mR^2\left[\dot{\phi}_1^2+\dot{\phi}_2^2+2\dot{\phi}_1\dot{\phi}_2\cos(\phi_1-\phi_2)\right]\\
      &\approx\frac{1}{2}mR^2(\dot{\phi}_1^2+\dot{\phi}_2^2+2\dot{\phi}_1\dot{\phi}_2)
   \end{align*}
   where in the last line I used the small angle approximation up to second order.
   
   The kinetic energy of the hoop is only rotational. The only "tricky" part is finding the right moment of inertia, but that's easy, because this is a 2D problem, so $I$ is basically a scalar. We just have to move it form the center to point $A$ by using the parallel axes theorem: $I=I_{\text{cm}}+mR^2=mR^2+mR^2=2mR^2$, so now our total kinetic energy is:
   \[T_{\text{tot}}=mR^2\dot{\phi}_1^2+\frac{1}{2}mR^2(\dot{\phi}_1^2+\dot{\phi}_2^2+2\dot{\phi}_1\dot{\phi}_2)\]
   
   To find the potential energy, all we need is the $y$ coordinate of the center of the hoop. This is given by $y_{\text{hoop}}=-R\cos\phi_1$, so our potential energy is \[U_\text{tot}=-2mgR\cos\phi_1-mgR\cos\phi_2\approx \frac{1}{2}mgR(2\phi_1^2+\phi_2^2)+U_0\]
   where, again, I approximated up to second order and put all the constants in $U_0$ since they won't show up in the equations of motion.
   
   After all the algebra with the Lagrangian, our equations of motion turn out to be:
   \begin{align*}
      3\ddot{\phi}_1+\ddot{\phi}_2&=-\frac{2g}{R}\phi_1\\
      \ddot{\phi}_1+\ddot{\phi}_2&=-\frac{g}{R}\phi_2
   \end{align*}
   which we can express in matrix notation by defining
   \[\mathbf{M}=
   \begin{pmatrix}
      3&1\\
      1&1
   \end{pmatrix},
   \hspace{1cm}
   \mathbf{K}= \frac{g}{R} \begin{pmatrix}
      2&0\\
      0&1
   \end{pmatrix}\]
   %\vspace{6cm}
   so now our equation is simply $\mathbf{M}\ddot{\boldsymbol{\phi}}=-\mathbf{K}\boldsymbol{\phi}$. The rest is the same old spiel: guess an exponential solution with frequency $\omega$, plug it back in, write in a form that necessitates a zero determinant, solve for $\omega$. So our equation for $\omega$ is:
   \[\text{det}\begin{pmatrix}
      2\frac{g}{R}-3\omega^2&-\omega^2\\
      -\omega^2&\frac{g}{R}-\omega^2
   \end{pmatrix}=(2\frac{g}{R}-3\omega^2)(\frac{g}{R}-\omega^2)-\omega^4=0\]
   This gives us values of $\omega$:
   \[\omega_1^2=\frac{2g}{R}, \hspace{1cm}\omega_2^2=\frac{g}{2R}\]
   which turns our $\mathbf{a}$ equation into, for $\omega_1$
   \[\begin{pmatrix}
      2\frac{g}{R}-6\frac{g}{R}&-2\frac{g}{R}\\
      -2\frac{g}{R}&\frac{g}{R}-2\frac{g}{R}
   \end{pmatrix}\cdot
   \begin{pmatrix} a_1\\a_2 \end{pmatrix}=-\frac{g}{R}
   \begin{pmatrix}
      4&2\\
      2&1
   \end{pmatrix}\cdot\begin{pmatrix}a_1\\a_2\end{pmatrix}=0\Rightarrow\mathbf{a}=
   \begin{pmatrix}1\\-2\end{pmatrix}\]
   and for $\omega_2$:
   
   \[\begin{pmatrix}
      2\frac{g}{R}-\frac{3g}{2R}&-\frac{g}{2R}\\
      -\frac{g}{2R}&\frac{g}{R}-\frac{g}{2R}
   \end{pmatrix}\cdot
   \begin{pmatrix} a_1\\a_2 \end{pmatrix}=\frac{g}{2R}
   \begin{pmatrix}
      1&-1\\
      -1&1
   \end{pmatrix}\cdot\begin{pmatrix}a_1\\a_2\end{pmatrix}=0\Rightarrow\mathbf{a}=
   \begin{pmatrix}1\\1\end{pmatrix}\]
   
   We can interpret the first solution as movements in opposite directions: the bead goes down when the hoop goes up, and viceversa. Also, the movement of the bead is twice as big as the movement of the loop. This makes some intuitive sense, since we imagine it's easier to move a point particle than an extended one.
   
   The other solution gives movements in phase and with the same amplitude. That is to say, the bead doesn't move with respect to the hoop.
   
\end{solution}

\question \begin{inparaenum}[\bfseries(a)] A simple pendulum of mass $M$ and length $L$ is suspended from a cart of mass $m$ that moves freely along a horizontal track. 

\item What are the normal frequencies?

\item Find and describe the corresponding normal modes
\end{inparaenum}

\begin{solution} 
   
   The coordinates I'll use are shown in the figure.
   \begin{center}
      \includegraphics[width=0.3\textwidth]{problem4}
      \captionof{figure}{The variables used in problem 4}
      \label{prob4fig}
   \end{center}
   
   From the figure we have
   \begin{align*}
      x_M=x_m+L\sin\phi &\Rightarrow \dot{x}_M=\dot{x}_m+L\dot{\phi}\cos\phi\\
      y_M=-L\cos\phi&\Rightarrow\dot{y}_M=L\dot{\phi}\sin\phi
   \end{align*}
   and let me call $x_m=x$ because I'll get tired really soon of writing the subscripts. With those relationships, and approximating the trig functions up to second order, the kinetic and potential energies become:
   \begin{align*}
      T&=T_m+T_M=\frac{1}{2}m\dot{x}^2+\frac{1}{2}M(\dot{x}_M^2+\dot{y}_M^2)\\
      &=\frac{1}{2}m\dot{x}^2+\frac{1}{2}M(\dot{x}^2+L^2\dot{\phi}^2+2L\, \dot{x}\dot{\phi}\cos\phi)\\
      &\approx\frac{1}{2}(m+M)\dot{x}^2+\frac{1}{2}M(L^2\dot{\phi}^2+2L\dot{x}\dot{\phi})\\
      U&= Mgy_M\\
      &=-MgL\,\cos\phi\\
      &\approx -MgL+\frac{1}{2}MgL\,\phi^2
   \end{align*}
   
   After calculating the Lagrange equations of motion we end up with:
   \begin{align*}
      (M+m)\,\ddot{x}+ML\ddot{\phi}&=0\\
      ML\,\ddot{x}+ML^2\ddot{\phi}&=-MgL\phi
   \end{align*}
   or, defining
   \[\mathbf{M}=\begin{pmatrix}
      M+m&ML\\
      ML&ML^2
   \end{pmatrix}, \hspace{1cm} \mathbf{K}=
   \begin{pmatrix}
      0&0\\
      0&MgL
   \end{pmatrix}\]
   we have $\mathbf{M\,\ddot{x}}=-\mathbf{K\,x}$. Now we can calculate all that stuff.
   
   \item The frequencies $\omega$ are, as usual, the eigenvalues of $\mathbf{M}^{-1}\mathbf{K}$, or the solutions of det$(\mathbf{K}-\omega^2\mathbf{M})=0$, which is the same. So:
   \[\text{det}
   \begin{pmatrix}
      -\omega^2(M+m)&-\omega^2ML\\
      -\omega^2ML&MgL-\omega^2ML^2
   \end{pmatrix}=-\omega^2ML(M+m)(g-\omega^2L)-\omega^4M^2L^2=0\]
   Working on that equation:
   \begin{align*}
      -\omega^2ML\left[g(M+m)-\omega^2L(M+m)+\omega^2ML\right]&=0\\
      -\omega^2 ML\left[g(M+m)-\omega^2Lm\right]&=0
   \end{align*}
   so our two normal frequencies are
   \[\omega_1=0\hspace{1cm}\omega_2=\sqrt{\frac{g(M+m)}{Lm}}\]
   
   \item To find the normal modes, we go back to $\mathbf{K}-\omega^2\mathbf{M}=0$ and plug in the values of $\omega$. The first one is easiest:
   \[\begin{pmatrix} 0&0\\0&MgL \end{pmatrix}\cdot
   \begin{pmatrix} a_1\\a_2 \end{pmatrix}=
   \begin{pmatrix} 0\\ a_2 \,MgL\end{pmatrix} =0\hspace{0.5cm}\Rightarrow\hspace{0.5cm} a_2=0\]
   
   Notice this doesn't tell us anything about $a_1$. However, $a_2$ represents $\phi$ in our case, so we can go back to our equations of motion and see what we get when $\phi(t)$ is identically zero. We can see this implies that $\ddot{x}=0$. That means that the cart is moving without acceleration, at a constant velocity. The pendulum doesn't move at all in this case, like we would expect, since it experiences no forces. We would need initial conditions to know more about the movement, but this is certainly a lot coming from a simple zero!
   
   Using our other value for $\omega$ we find:
   \[\begin{pmatrix}
      -g\frac{(M+m)^2}{Lm} & -g(M+m)\frac{M}{m}\\
      -g(M+m)\frac{M}{m} & MgL-gL(M+m)\frac{M}{m}
   \end{pmatrix}\cdot
   \begin{pmatrix}a_1\\a_2\end{pmatrix}=0\]
   So we get an equation that relates $a_1$ to $a_2$:
      \[(M+m)a_1+MLa_2=0\hspace{0.5cm}\Rightarrow\hspace{0.5cm}a_1=-\frac{ML}{M+m}a_2\]
   
   Now, if you recall, this whole $a$ business came about by assuming oscillatory motion, so the fact we get a solution means there is, in fact, oscillatory motion. The frequency of the motion is given by $\omega_2$. As we would expect, this is the frequency of a simple pendulum modified by the two masses. Note that if $m\rightarrow\infty$ then the $M$ in the numerator of $\omega_2$ doesn't matter and we can cancel the $m$'s, recovering the simple pendulum frequency. This is to be expected, since an infinitely massive cart wouldn't move.
   
   Also, the amplitudes of the oscillations of the pendulum and the cart are related via the relationship between $a_1$ and $a_2$. Notice first of all that they are oscillating out of phase, due to the minus sign. Also, $a_1\rightarrow0$ when $m\rightarrow\infty$, as we would expect. 
   
\end{solution}

\question \begin{inparaenum}[\bfseries(a)] As a model of a linear triatomic molecule such as CO$_2$, consider a system with two identical atoms each of mass $m$ connected by two identical springs to a single atom of mass $M$. To simplify matters, assume that the system is confined to move in one dimension.

\item Write down the Lagrangian and find the normal frequencies of the system. Show that one of the normal frequencies is zero.

\item Find and describe the motion in the normal modes whose frequencies are nonzero.

\item Do the same for the mode with zero frequency.
\end{inparaenum}

\begin{solution}
\begin{inparaenum}[\bfseries(a)]
   
   \item Since the system can only move in one dimension, we'll have just one coordinate for each atom. Call them $x_1$, $x_2$ and $x_3$. Now, the kinetic energy is easy enough: $T=\frac{1}{2}(m\dot{x}_1^2+m\dot{x}_3^2+M\dot{x}_2^2)$, but the potential energy is slightly more complicated, because the springs stretch and compress due to the different molecules. But it's not so bad. Basically, motion of the middle molecule to the right will compress the right spring and stretch the left one, so we have to add $x_2$ to one of the coordinates and subtract it from the other one. It doesn't really matter which. I've chosen to make it so that $U=\frac{1}{2}k[(x_2-x_1)^2+(x_3-x_2)^2]$
   
   With that, the Lagrangian becomes \[\mathcal{L}=\frac{1}{2}\left[m(\dot{x}_1^2+\dot{x}_3^2)+M\dot{x}_2^2\right]-\frac{1}{2}k\left[(x_2-x_1)^2+(x_3-x_2)^2\right]\] and our equations of motion are:
   \begin{align*}
      m\ddot{x}_1&=k(x_2-x_1)\\
      M\ddot{x}_2&=k(x_3+x_1)-2kx_2\\
      m\ddot{x}_3&=-k(x_3-x_2)
   \end{align*}
   which leads us to define
   \[\mathbf{M}=
   \begin{pmatrix}
      m&0&0\\
      0&M&0\\
      0&0&m
   \end{pmatrix}, \hspace{1cm}
   \mathbf{K}=k \begin{pmatrix}
      1&-1&0\\
      -1&2&-1\\
      0&-1&1
   \end{pmatrix}\]
   Thus, we now have the equation:
   \[\text{det}\begin{pmatrix}
      k-\omega^2m&-k&0\\
      -k&2k-\omega^2M&-k\\
      0&-k&k-\omega^2m
   \end{pmatrix}= (k-\omega^2m)[(2k-\omega^2M)(k-\omega^2m)-k^2]-k^2(k-\omega^2m)=0\]
   which we can re-write as \[\omega^2\left[-\omega^4m^2M+2km(M+m)\omega^2-k^2(2m+M)\right]=0\] so we can see that we do get a zero frequency, as advertised. Now we can also get the other two frequencies. Solving the equation with Mathematica we get: 
   \begin{align*}
      \omega_1&=\pm\sqrt{\frac{k}{m}}\\
      \omega_2&=\pm\sqrt{k\left(\frac{1}{m}+\frac{2}{M}\right)}
   \end{align*}
   
   \item Now we want to find the values of $\mathbf{a}$. Our equations are, for the first $\omega_1$:
   \[\begin{pmatrix}
      0&-k&0\\
      -k&2k-kM/m&-k\\
      0&-k&0
   \end{pmatrix}\cdot\begin{pmatrix}a_1\\a_2\\a_3\end{pmatrix}=k
   \begin{pmatrix}
      -a_2\\
      -a_1+(2-M/m)a_2-a_3\\
      -a_2
   \end{pmatrix}=0\]
   So we find that $a_2=0$ and $a_1=-a_3$. This means that the carbon molecule is not moving and the two oxygen molecules are moving in opposite directions. The center of mass of the molecule is not moving, like we would expect.
   
   For $\omega_2$ we get:
   \[\begin{pmatrix}
      -2km/M&-k&0\\
      -k&-k/m&-k\\
      0&-k&-2km/M
   \end{pmatrix}\cdot\begin{pmatrix}a_1\\a_2\\a_3\end{pmatrix}=-k
   \begin{pmatrix}
      2a_1\,m/M+a_2\\
      a_1+a_2/m+a_3\\
      a_2+2a_3m/M
   \end{pmatrix}=0\]
   which means that $a_1=a_3$ and $a_2=-2a_1\,m/M$. That is, in this case, the two oxygens are moving in phase and with the same amplitudes, and the carbon is moving opposite them and with an amplitude determined by the ratio of the masses.
   
   \item Using $\omega=0$ our equation is simply
   \[k\begin{pmatrix}
      1&-1&0\\
      -1&2&-1\\
      0&-1&1
   \end{pmatrix}\cdot\begin{pmatrix}a_1\\a_2\\a_3\end{pmatrix}=k
   \begin{pmatrix}
      a_1-a_2\\
      -a_1+2a_2-a_3\\
      -a_2+a_3
   \end{pmatrix}=0\]
   which means that $a_1=a_2=a_3$. This, together with the fact that the frequency of the motion is zero, tells us that this describes a displacement of the entire molecule rather than a vibration of it.
   
   
\end{inparaenum}
\end{solution}

\end{questions}
\end{document}

Edit: I do know that this isn't an optimal solution, but in some cases it may be sufficient.
yaelcita
Posts: 6
Joined: Sat May 29, 2010 11:46 pm

Re: My page breaks are being ignored!

Post by yaelcita »

Ok, that works (changing the floats to just a centered figure). I have no idea why my floats were causing problems, though... I had never had this happen before. Could it be related to the image file?

Thanks everyone for all your help!!
Post Reply